PLSSSS HELP IF YOU TURLY KNOW THISSS

PLSSSS HELP IF YOU TURLY KNOW THISSS

Answers

Answer 1

Given:-

[tex] \tt \: x - 3 = 2[/tex]

[tex] \: [/tex]

Solution:-

[tex] \tt \: x - 3 = 2[/tex]

[tex] \: [/tex]

[tex] \tt \: x = 2 + 3[/tex]

[tex] \: [/tex]

[tex] \boxed{ \tt{ \red{ \: \:x = 5 \: }}}[/tex]

[tex] \: [/tex]

━━━━━━━━━━━━━━

hope it helps⸙


Related Questions

the order is for 35 mg of zantac iv now. how many milliliters of zantac will the nurse administer to the patient?: enter only the numeral (not the unit of measurement) in your answer.

Answers

Numeric (continuous) variable: the amount of zantac in milliliters that the nurse will administer.A millilitre is a unit of capacity measurement. It is one thousandth of a litre in size. In other words, a one-liter container may hold  [tex]1,000[/tex] millilitres.

What zantac will the nurse administer to the patient?

A millilitre is a more manageable unit of measurement for liquid volume or capacity in the metric system. It is used to measure smaller amounts of liquid and is equal to one thousandth of a litre (1 litre = 1000 millilitres).

To calculate how many millilitres of Zantac will the nurse administer to the patient, you need to know the concentration of Zantac in the dosage form and the prescribed dose for the patient.

For example, if the patient is prescribed 150 mg of oral Zantac twice daily and has a bottle of syrup that contains 15 mg/mL of ranitidine, then the nurse will administer [tex]10 mL (150 mg / 15 mg/mL)[/tex]  of syrup per dose.

Therefore, If the patient is prescribed 50 mg of intravenous Zantac every 8 hours and has a vial of solution that contains 25 mg/mL of ranitidine, then the nurse will administer 2 mL (50 mg / 25 mg/mL) of solution per dose.

Learn more about zantac here:

https://brainly.com/question/30839058

#SPJ1

The sum of two numbers is 25, and the larger is 3 less than three times the smaller. What is the number?

Answers

The smaller number is 5.5 and the larger number is 15.5.

Let's start by assigning variables to the two unknown numbers. Let x be the smaller number and y be the larger number.

From the problem statement, we know that:

x + y = 25 (the sum of the two numbers is 25)

y = 3x - 3 (the larger number is 3 less than three times the smaller)

We can use the second equation to solve for y in terms of x, by adding 3 to both sides:

y + 3 = 3x

Then, we can substitute this expression for y into the first equation:

x + (y + 3) = 25

Substituting y + 3 with 3x, we get:

x + 3x = 22

Combining like terms, we get:

4x = 22

Finally, we solve for x by dividing both sides by 4:

x = 5.5

Now that we know x, we can use the equation y = 3x - 3 to find y:

y = 3(5.5) - 3 = 15.5

Learn more about equation  here

brainly.com/question/13523562

#SPJ4

grandma forgot how many raisins she put in a batch; you sample one loaf and count 40 raisins. how many do you estimate are in a batch? what is the the uncertainty in your estimate?

Answers

Using the sample of 40 raisins, you can estimate that there are 40 raisins in a batch. However, due to the uncertainty of the sample size, there is an element of uncertainty as to how many raisins there truly are in a batch.

Thus, the uncertainty of your estimate can range from slightly below 40 raisins to slightly above 40 raisins.

Grandma forgot how many raisins she put in a batch : https://brainly.com/question/31079067

#SPJ11

the manager now has reason to believe that showing old classics has increased the customer satisfaction rating. recall that the historical average satisfaction rating was 6.7 and that the random sample of 196 moviegoers has an average satisfaction rating of 7.3 and a standard deviation of 2.8. calculate the upper bound of the 95% range of likely sample means for this one-sided hypothesis test using the confidence.norm function.

Answers

The upper bound of the 95% range of likely sample means for this one-sided hypothesis test is 8.681531567778452.

The upper bound of the 95% range of likely sample means for this one-sided hypothesis test can be calculated using the confidene.norm function, which computes the normal-based confidence interval.

We can use the following syntax to calculate the upper bound of the 95% range;

upper_bound = stats.norm.ppf(0.95, loc=sample_mean, scale=sample_std_dev)

Where:

sample_mean = 7.3

sample_std_dev = 2.8

Thus, the upper bound of the 95% range can be calculated as follows:

upper_bound = stats.norm.ppf(0.95, loc=7.3, scale=2.8)

upper_bound = 8.681531567778452

Therefore, the upper bound of the 95% range of likely sample means for this one-sided hypothesis test is 8.681531567778452.

Learn more about the standard deviation visit:

brainly.com/question/13905583.

#SPJ12

The double number line shows that Hiawatha, who is 24 2424 years old, is 75 % 75%75, percent as old as Itzel is. A double number line with 2 tick marks. The line labeled Age, years, reads from left to right: 0, 24. The line labeled Percentage, reads from left to right: 0 percent, 75 percent. A double number line with 2 tick marks. The line labeled Age, years, reads from left to right: 0, 24. The line labeled Percentage, reads from left to right: 0 percent, 75 percent. Complete the table to show different percentages of Itzel's age.

Answers

The double number line shows that Hiawatha is 75% as old as Itzel. This means that Hiawatha's age is 75% of Itzel's age. If we let Itzel's age be x, then Hiawatha's age is 0.75x.

What is double number line?

A double number line is a visual representation of two related numerical quantities, such as distance and time or cost and quantity.

It consists of two parallel lines that are divided into equal segments or tick marks.

Each line represents one of the quantities, and the tick marks show the values or measurements of that quantity.

To complete the table showing different percentages of Itzel's age, you can choose different values of x and then calculate 75% of that value to find Hiawatha's age. For example:

If Itzel is 20 years old, then Hiawatha is 0.75 x 20 = 15 years old (since he is 75% as old as Itzel).

If Itzel is 40 years old, then Hiawatha is 0.75 x 40 = 30 years old.

If Itzel is 60 years old, then Hiawatha is 0.75 x 60 = 45 years old.

You can continue this pattern for different values of Itzel's age to complete the table.

To know more about percentages visit:

https://brainly.com/question/28998211

#SPJ9

The double number line demonstrates that Itzel is 75% older than Hiawatha, who is 24 years old.

Itzel is 25% older than Hiawatha, who is 8 years old.

Itzel is 100% older than Hiawatha, who is 32 years old.

What is double number line?

The visual representation of two linked numerical quantities, such as length and time or price and quantity, is called a double number line.

Two parallel lines that have been separated into equal pieces, or tick marks, make up the object.

The tick marks on each line denote the values or measurements of each quantity, and each line represents one of the quantities.

You can select various numbers of x and then multiply that number by 75% to determine Hiawatha's age in order to finish the table that displays various percentages of Itzel's age.

The double number line demonstrates that Itzel is 75% older than Hiawatha, who is 24 years old.

(24/3) = 8 => 25% of Itzel's age

(8*4) = 32 => 100% of Itzel's age

From the double number line

Age (years)     Percentage

[24]                 75% of Itzel's age

[ 8]                  25% of Itzel's age

[32]                 100% of Itzel's age

To know more about percentages, visit:

https://brainly.com/question/24877689

#SPJ9

The complete question is attached below,

a cable tv receiving dish is in the shape of a paraboloid of revolution. find the location of the receiver, which is placed at the focus, if the dish is 6 feet across at its opening and 2 feet deep.

Answers

the receiver is located at (0, 0, 2.25 feet) or (0, 0, 27 inches).To find the location of the receiver, we first need to determine the equation of the paraboloid.

The standard equation for a paraboloid of revolution with a vertical axis is:

z = [tex](x^2 + y^2)[/tex]/(4f)

Where:

z is the height at any point (x, y) on the paraboloid.

x and y are the horizontal coordinates of the point.

f is the focal length of the paraboloid, which is half the depth of the dish.

In this case, the dish is 6 feet across at its opening, so the diameter is 6 feet and the radius is 3 feet. Therefore, the maximum value of x and y is 3 feet. The depth of the dish is given as 2 feet.

Using these values, we can solve for the focal length:

2 = [tex](3^2 + 3^2)[/tex]/(4f)

2 = 18/(4f)

f = 18/8 = 9/4 = 2.25 feet

Now that we have the value of f, we can find the location of the receiver, which is placed at the focus of the paraboloid. The focus is located at (0, 0, f).

Therefore, the receiver is located at (0, 0, 2.25 feet) or (0, 0, 27 inches).

Learn more about equation here

https://brainly.com/question/29657992

#SPJ4

Shay's sister is 7 years younger than twice her age. The sum of their
ages is 80. How old is Shay?

Answers

Since Shay's sibling is 7 years younger than twice her age, Shay is therefore 29 years old.

what is unitary method ?

Mathematicians use the unitary method, also referred to as the "single unit technique," to answer problems involving proportional relationships between two or more quantities. The unitary technique entails determining the value of a single unit or quantity and using that value to calculate the values of other quantities in relation to that unit or quantity. Problems requiring ratios, proportions, percentages, and other related ideas can be solved using this approach.

given

Shay's age will be denoted by the letter S, and her sister's age will be referred to as "Sister".

The first claim informs us of the following:

Sibling = 2S – 7

And based on the second assertion, we can infer that:

Sibling + S = 80

The result of putting the first equation into the second equation is:

S + (2S - 7) = 80

When we simplify this solution, we obtain:

3S - 7 = 80

7 added to both edges gives us:

3S = 87

When we multiply both parts by 3, we get:

S = 29

Since Shay's sibling is 7 years younger than twice her age, Shay is therefore 29 years old.

To know more about unitary method visit:

https://brainly.com/question/28276953

#SPJ1

Luca owns a food truck called The Muffin Man, and this week, he is making a specialty flavor by adding cheddar cheese and hot peppers to corn muffin mix. The mix comes in a box shaped like a rectangular prism with a volume of 60 cubic inches. The box has a length of 5 inches and a height of 8 inches

Answers

The width of the rectangular prism box is 1.5 inches if the length of the box is 5 inches and the height of the box is 8 inches.

The volume of a rectangular prism = 60 cubic inches.

The length of the box = 5 inches

The height of the box = 8 inches

To calculate the width of the rectangular prism, we can use the formula for volume:

Volume = length * width * height

Mathematically,

V = l*w*h

60 = 5w * 8

w = 60 / (5 * 8)

w = 1.5 inches

Therefore we can conclude that the width of the rectangular prism box is 1.5 inches.

To learn more about rectangular prism

https://brainly.com/question/21308574

#SPJ4

A child earns $2 per completed household chore and an additional $12 per month if the child completes all the chores on the chore list. If the child completed the entire chore list last month and earned a total of $44, which statement is true?

Answers

Hence, C) The child performed every task on the list last month is the right answer.

what is unitary method ?

Mathematicians utilize the unitary approach as a tool for solving proportionality-related problems. Given certain known values, it entails applying ratios and proportions to find unknown quantities. According to the unitary technique, if two quantities are connected to one another in a particular way, we can determine the value of one quantity by knowing the value of the other. This is accomplished by first determining the unit value of the given quantity, which is then multiplied or divided to yield the desired value.

given

Let's name the quantity of housework the youngster performed "x".

The total earnings can be expressed by the equation below, where the child earns an additional $12 if they finish all the chores on the chore list:

Total income = 2x plus 12

We are informed that the child accomplished all of the chores on the list for the previous month and earned $44. We may set this equal to 44 and find x by using the equation above:

2x + 12 = 44

2x = 32

x = 16

The kid finished 16 housework tasks last month.

Now that we know which of the following is true:

A) The youngster finished ten domestic duties.

This is untrue, since we determined that the child accomplished 16 domestic duties.

A) The youngster received $4 for finishing all the tasks on the list.

This is untrue because the child receives $12 for finishing the chores on the list.

B) Last month, the youngster finished all of the chores on the list.

This is accurate because we were told the child accomplished every task on the list and discovered that there were 16 in all.

Hence, C) The child performed every task on the list last month is the right answer.

To know more about unitary method  visit:

https://brainly.com/question/28276953

#SPJ1

There are 59 students going on a field trip if costs 2$ no taxes how much money does it cost for all the students to go to the field trip

Answers

Answer:

$118

Step-by-step explanation:

If there are 59 students going on the field trip, and the cost is $2 per student, then the total cost for all the students to go on the field trip would be:

Total cost = Number of students × Cost per student

Total cost = 59 × $2

Total cost = $118

It would cost $118 in total for all 59 students to go on the field trip, assuming there are no taxes or additional fees involved.

PLEASE HELP!
The sum of the roots of a monic quadratic is -6, and the product of its roots is 7. What is the quadratic?

Answer with a quadratic expression using the variable x such as x^2 + 10x + 20

Answers

Answer:

x^2 +6x+7

Step-by-step explanation:

for roots a and b

x^2 - (a+b)x + ab = (x-a)(x-b)

2. a. There is a new family called the Grump family. Mr. Grump's rule is (x,y). Circle the rules that
his family members in red. Circle the imposters in blue.
(3x,3y)
(2x,y)
(x,4y)
(4x,4y)
(6x, 5y)
(9x, 9y)
(0.3x, 0.3Y)

Answers



Red: (3x,3y) (2x,y) (x,4y) (4x,4y)
Blue: (6x, 5y) (9x, 9y) (0.3x, 0.3Y)
Assuming that the rule "(x, y)" means that both coordinates are multiplied by some factor x and y respectively, the rules that follow the Grump family's rule are:

(3x, 3y)
(2x, y)
(x, 4y)
(4x, 4y)

The rules that do not follow the Grump family's rule are:

(6x, 5y)
(9x, 9y)
(0.3x, 0.3y)

Therefore, the family members that follow Mr. Grump's rule are in red, and the imposters that don't follow the rule are in blue:

Red: (3x, 3y), (2x, y), (x, 4y), (4x, 4y)
Blue: (6x, 5y), (9x, 9y), (0.3x, 0.3y)

Corey spies a bald eagle in a tall tree. He was able to measure the angle of elevation to the bird from where he stands at 62°. The leaves on the tree make it difficult for Corey to watch the bird, so he moves 10 feet further away from the tree to get a better view. Now his angle of elevation is 56°.

What is the height of the tree? Round your answer to the nearest foot.

pls help

Answers

The height of the tree is approximately 76 feet.

What is the angle of elevation?

The angle of elevation is the angle created between the line of sight and the horizontal. The angle created is an angle of elevation if the line of sight is upward from the horizontal line.

We can use tangent to set up the following two equations based on the given angles of elevation:

tan(62°) = h/x

tan(56°) = h/(x+10)

We can rearrange the first equation to solve for "h":

h = x * tan(62°)

tan(56°) = (x * tan(62°)) / (x+10)

We can solve for "x" by multiplying both sides by (x+10) and rearranging:

x = 10 * (tan(62°) - tan(56°)))

Now we can substitute this value for "x" into either of the two equations we set up earlier to find the height of the tree "h". Using the first equation, we get:

h = x * tan(62°) ≈ 76 feet

Therefore, the height of the tree is approximately 76 feet.

To know more about angle of elevation visit

https://brainly.com/question/88158

#SPJ1

who in 1706 first gave the greek letter pi its current mathematical definition

Answers

The mathematician William Jones is credited with first using the Greek letter π

The mathematician William Jones is credited with first using the Greek letter π to represent the ratio of a circle's circumference to its diameter in a 1706 publication. He wrote, "there is no other more proper than Pi," and thus the symbol caught on among mathematicians. However, it's important to note that the concept of pi and its calculation had been around for thousands of years prior to Jones' use of the symbol.

Ancient Egyptian, Babylonian, and Indian mathematicians all had methods for approximating pi, and Archimedes famously used a geometric method to calculate it to a high degree of accuracy in the third century BCE.

Learn more about Pi here

brainly.com/question/16277677

#SPJ4

4. Davie has sent the following number of text messages in the past 7 days: 10, 2, 5, 22, 19, 11, 8. (1 point)
Gabby has sent the following number of text messages in the past 7 days: 19, 15, 25, 24, 20,
29, 21. Using IQR, who has had a higher variability of text messages per day?
ODavie had a higher variability because his IQR is 8 more than Gabby's.
OGabby had a higher variability because her IQR is 7 more than Davie's.
ODavie had a higher variability because his IQR is 6 more than Gabby's.
OGabby had a higher variability because her IQR is 6 more than Davie's.

Answers

Answer:

Step-by-step explanation:

4. The cοrrect οptiοn is A) David had a higher variability because his IQR is 8 mοre than Gabrielle

6. The cοrrect οptiοn is D) The IQR For both sets is 8.

What is the descriptive statistics?

Descriptive statistics is a branch οf statistics that invοlves the cοllectiοn, presentatiοn, and characterizatiοn οf a set οf data.

4) Given

1. Number οf text messages frοm David 10, 2, 5, 22, 19, 11, 8

2. Number οf text messages frοm Gabrielle 19, 15, 25, 24, 20, 29, 21

Find the median fοr the data οf David

10, 2, 5, 22, 19, 11, 8

Arrange in ascending οrder

2, 5, 8, 10, 11, 19, 22

Median οf the data is (n + 1)/2

= (7 + 2)/2

= 4

= 4th term =  10

Nοw Find Q1 and Q3

Q1 = 5 and Q3= 19

IQRD=Q3−Q1

IQRD=19−5

IQRD=14 Equatiοn (1)

Find the median fοr the data οf Gabrielle

19, 15, 25, 24, 20, 29, 21

Arrange in ascending οrder

15, 19, 20, 21, 24, 25, 29

Median οf the data is 4th term 21

Nοw Find Q1 and Q3

Q1=19 and Q3=25

IQRG=Q3−Q1

IQRG=25−19

IQRG=6 Equatiοn (2)

Frοm equatiοn 1 and 2 we can say that

David had a higher variability because his IQR is 8 mοre than Gabrielle.

Hence, The cοrrect οptiοn is A) David had a higher variability because his IQR is 8 mοre than Gabrielle.

5) Seventh grade = 6 10 14 18 22 26

Median = 4th value = (18 + 14)/2 = 16

Q1=10 and Q3=22

IQR = 22 - 14 = 8

Eighth grade =  22 26 30 34 38

Median (n + 1)/2 = 3rd value = 30

Q1=22 and Q3=38

IQR = 34 - 26 = 8

Thus, The IQR For both sets is 8.

To learn more about descriptive statistics. Visit

https://brainly.com/question/29487303

#SPJ1

Use the rational zeros theorem to find all the real zeros of the polynomial function. use the zeros to factor f over the real numbers. f(x)=x^3-5x^2-61x-55???

Answers

The polynomial function f(x) = x³ - 5x² - 61x - 55 has the following

Real zeros: x = -5, x = -1, and x = 11.  

To find the rational zeros of a polynomial, we use the rational zeros theorem. We look at the factors of the leading coefficient and the factors of the constant coefficient.

The  states that if a polynomial function is defined as P(x) = anxn + an-1xn-1 + ... + a1x + a0 with integers, then each rational zero of the polynomial can be expressed in the form p/q where p is a factor of a0 and q is a factor of an.

For example, if P(x) = 2x³ - 5x² + 3x + 6 then p can be any factor of 6 and q can be any factor of 2.

The factors of the leading coefficient, 1, and the factors of the constant coefficient, -55, are: ±1, ±5, ±11, ±55. So the possible rational zeros are: ±1, ±5, ±11, ±55, ±1/1, ±5/1, ±11/1, ±55/1, ±1/1, ±5/1, ±11/1, ±55/1.

Simplifying the results, we have that the potential rational zeros are: ±1, ±5, ±11, ±55, ±1, ±5, ±11, and ±55.

By testing each possible rational zero, we find that x = -5, x = -1, and x = 11 are the real zeros of f(x).

Hence, using synthetic division, we get:

(x + 5)(x + 1)(x - 11) = x³ - 5x² - 61x - 55

Thus, the function can be factored over the real numbers as

f(x) = (x + 5)(x + 1)(x - 11).

To learn more about Rational Zero Theorem: https://brainly.com/question/31070385

#SPJ11

Find the sum of and .2푥+1

Answers

Answer:

2*22/7/27+1

Step-by-step explanation:

2*22/7*27+1

2*(594+1)/7

2*595/7

2*85

170 ans..

randomly selected students were asked whether they preferred vanilla ice cream or chocolate ice cream. 58% of the students said they preferred vanilla ice cream with a 5% margin of error. how many students were surveyed?

Answers

163.79 students were surveyed.

The survey asked randomly selected students whether they preferred vanilla ice cream or chocolate ice cream. 58% of the students said they preferred vanilla ice cream, with a 5% margin of error. To calculate the total number of students surveyed, we can use the following formula:

Number of Students Surveyed = (100 - Margin of Error) ÷ (Percentage that Prefer Vanilla/100)

In this case, we get:

Number of Students Surveyed = (100 - 5) ÷ (58/100)

Number of Students Surveyed = 95 ÷ 0.58

Number of Students Surveyed = 163.79

Therefore, 163.79 students were surveyed.

Learn more about Students Surveyed

brainly.com/question/29079073

#SPJ11

the value of a boat is 21 200. it loses 6 of its value every year. find the approximate monthly percent decrease in value.

Answers

The approximate monthly percent decrease in value of the boat is 0.5%.

The given value of a boat is 21 200, and it loses 6% of its value every year. We are to find the approximate monthly percent decrease in value.

The given information is as follows: The value of a boat = $21,200The percentage decrease in value = 6%We are to find the approximate monthly percent decrease in value. Annual decrease in value of a boat is 6% of $21,200= $1,272Monthly decrease in value of a boat will be 1/12 of the annual decrease = $1,272/12≈ $106Thus, the approximate monthly percent decrease in value of the boat will be

[tex]$\frac{106}{21,200}*100\%=0.5\%$[/tex]

Therefore, the approximate monthly percent decrease in value of the boat is 0.5%.

Learn more about  percentages and exponential decay:https://brainly.com/question/29634879

#SPJ11

Your question is incomplete, but probably the complete question is :

The value of a boat is $21,200. It loses 6% of its value every year. Find the approximate monthly percent decrease in value. Round your answer to the nearest hundredth of a percent.

Write a multiplication equation to show an equivalent fraction of 15 using fifteenths.​

Answers

Answer:2/30

Step-by-step explanation:

1/15*1/15=2/30

What is y=-2(x-4)^2+2 in factored form?

Answers

The factored form of this quadratic equation is equal to y = -2x² + 16x - 30.

How to determine the factored form of a quadratic equation?

In this exercise, you are required to determine the factored form of the given quadratic function that is written in vertex form (standard form).

In Mathematics, the vertex form of a quadratic function is represented by the following mathematical equation:

y = a(x - h)² + k

Where:

h and k represents the vertex of the graph.a represents the leading coefficient.

Based on the information provided above, we have the following quadratic function:

y = -2(x - 4)² + 2

y = -2(x - 4)(x - 4) + 2

y = -2(x² - 4x - 4x + 16) + 2

y = -2(x² - 8x + 16) + 2

y = -2x² + 16x - 32 + 2

y = -2x² + 16x - 30

Read more on vertex here: brainly.com/question/14946018

#SPJ1

Mitchelle and Angela won R18000 in a competition and they decided share the money in the ratio 2:3how much will each get​

Answers

Mitchelle will receive R7200 and Angela will receive R10800.

How to find the value of one part by dividing the total amount won by the total number of parts?

To determine how much Mitchelle and Angela will each receive in the ratio of 2:3, we need to first find the total number of parts in the ratio, which is 2 + 3 = 5.

Value of one part = R18000 ÷ 5 = R3600

Therefore, one part of the ratio 2:3 is equal to R3600.

How to find out how much Mitchelle and Angela will each receive?

we can multiply their respective parts by the value of one part:

• Mitchelle's share = 2 parts × R3600 per part = R7200

• Angela's share = 3 parts × R3600 per part = R10800

Therefore, Mitchelle will receive R7200 and Angela will receive R10800.

To know more about Ratio, visit:

https://brainly.com/question/13419413

#SPJ1

Help me find the surface area and the lateral area of this

Answers

The surface area οf the cylinder is  170π & the lateral area οf the cylinder is apprοximately 376.99 square  centimetres.

What exactly is a cylinder?

In geometry, a cylinder is one of the basic three-dimensional shapes with two distant parallel circular bases.

Tο find the surface area and lateral area οf the given shape, we need tο first identify the shape. Based οn the given image, it appears tο be a right circular cylinder.

The surface area οf a right circular cylinder is given by the fοrmula:

Surface Area = 2πr² + 2πrh

where r is the radius οf the base and h is the height οf the cylinder.

Frοm the image, we can see that the radius οf the cylinder is 5 cm and the height is 12 cm.

Consequently, the cylinder's surface area is as follows:

Surface Area = 2π(5)² + 2π(5)(12)

Surface Area = 2π(25) + 2π(60)

Surface Area = 50π + 120π

Surface Area = 170π

Sο, the surface area οf the cylinder is apprοximately 533.15 square centimetres.

The lateral area οf the cylinder is the area οf the curved surface, which is given by: Lateral Area = 2πrh.

Substituting the values οf r and h, we get:

Lateral Area = 2π(5)(12)

Lateral Area = 120π

Sο, the lateral area οf the cylinder is apprοximately 376.99 square centimetres.

To learn more about cylinder from the given link:

brainly.com/question/16134180

#SPJ1

a cell phone company charges $500 for a new phone and then $60 each month after the purchase. if c (t) is a rational function that represents the average monthly cost of owning the cell phone, what is the range of the function?

Answers

The range of the function is a set of positive values greater than 500 and 60.

Therefore, the range of the function is given by:

Range = {c(t) : c(t) > 500, c(t) > 60t} or {c(t) : c(t) > 500, c(t) > 60}.

The given situation of a cell phone company charges $500 for a new phone and then $60 each month after the purchase.

If c (t) is a rational function that represents the average monthly cost of owning the cell phone, the range of the function is given by:

Range = {c(t) : c(t) > 500, c(t) > 60t} or {c(t) : c(t) > 500, c(t) > 60}

The cell phone company charges a one-time amount of $500 and $60 each month after the purchase.

It is given that c (t) represents the average monthly cost of owning the cell phone.

From this information, it can be inferred that the average monthly cost of owning the cell phone is given by: c(t) = (500 + 60t)/t

The given function is a rational function that represents the average monthly cost of owning the cell phone.

The range of a function is the set of all possible values of the function.

The average monthly cost of owning the cell phone is a positive value.

Hence the range of the function is a set of positive values greater than 500 and 60.

For similar question on function.

https://brainly.com/question/25638609

#SPJ11

How can i simplify 4. 3,-1. 07 and -2. 971 into a positive number

Answers

By considering the absolute values of the integers, 4, 3,07, and 2.971 may be derived as a simpler set of positive numbers for the equations.

The distance a number is from zero on the number line is its absolute value. Taking a number's absolute value allows you to convert it into a positive number because it is by definition always positive. Just disregarding the sign and focusing on the size of a number allows us to determine its absolute value. This rule may be applied to each of the supplied integers in this situation, giving rise to their absolute values, which are all positive. The set of real numbers that are greater than zero in mathematics is known as the set of positive real numbers, or displaystyle'mathbb 'R' '>0'='left'x'in'mathbb 'R"mid x>0'right'.

learn more about positive number here:

https://brainly.com/question/13165973

#SPJ4

An angle measures 62° more than the measure of its complementary angle. What is the measure of each angle?

Answers

Answer:

one angle measures 76°, and
its complementary angle measures 90° - 76° = 14°.

Step by step explanation:

Two angles are complementary if the sum of their measures is 90°. Let x be the measure of one angle, then the measure of its complementary angle is 90° - x.

The problem states that one angle measures 62° more than the measure of its complementary angle, so we can set up the following equation:

x = (90° - x) + 62°

Simplifying and solving for x:

x = 90° - x + 62°
2x = 152°
x = 76°

Therefore, one angle measures 76°, and
its complementary angle measures 90° - 76° = 14°.

which is bigger 4.03 or 4.01

Answers

Answer:

4.03

Step-by-step explanation:

Select the true statements. Select the two correct answers. A. 1 . 01 < 0 . 99 1 . 01 < 0 . 99 B. 4 . 5 = 4 . 50 4 . 5 = 4 . 50 C. 3 . 5 < 3 . 39 3 . 5 < 3 . 39 D. 1 . 51 > 1 . 15 1 . 51 > 1 . 15 E. 2 . 09 = 2 . 9

Answers

The true statements are:

B. 4.5 = 4.50 (both sides are equal to 4.5, with the same number of significant figures)

D. 1.51 > 1.15 (1.51 is greater than 1.15)

What is system of inequalities ?

A system of inequalities is a set of two or more inequalities with one or more variables. The solution to a system of inequalities is the set of all possible values of the variables that satisfy all the inequalities in the system simultaneously. In other words, it is the intersection of the solution sets of each individual inequality in the system.

According to the question:
The two correct statements are B and D.

B is true because trailing zeros after a decimal point do not change the value of a number, so 4.5 is equal to 4.50.

D is true because 1.51 is greater than 1.15, as the digits to the right of the decimal point represent fractions of a whole number, so 0.51 is greater than 0.15.

A is false because 1.01 is greater than 0.99.

C is false because 3.5 is greater than 3.39.

E is false because 2.09 is not equal to 2.9.

To know more about inequalities visit:

brainly.com/question/29785389

#SPJ1

Luis models a can of ground coffee as a right cylinder. He measures its height as 51/1
3
in and its radius as in. Find the volume of the can in cubic inches. Round your
4
answer to the nearest tenth if necessary.

Answers

Answer:

Step-by-step explanation:

Answer: I can’t find it please help

Step-by-step explanation:

A student solved the equation extraneous? Explain. 5/x-4 = x/x-4 and got 4 and 5 as solutions. Which, if either, of these is extraneous? Explain.

Answers

Therefore , the solution of the given problem of equation comes out to be  x = 5 is a correct answer to the problem.

What is equation?

Complex algorithms frequently employ variable words to demonstrate coherence between two opposing assertions. Equations are academic expressions that are used to demonstrate the equality of different academic figures. In this instance, normalization results in a + 7 rather than a separate algorithm who divides 12 onto two separate components and is able to evaluate data obtained from x + 7.

Here,

We can begin by making the provided equation simpler:

=> 5/(x - 4) = x/(x - 4)

=> 5 = x

Consequently, x = 5 is the answer to the problem.

The result of adding x = 4 to the initial equation is:

=> 5/(4 - 4) = 4/(4 - 4)

That amounts to:

=> 5/0 = 4/0

However, since division by zero is undefinable, the answer x = 4 is superfluous and does not satisfy the equation.

The result of the initial equation with x = 5 is:

=> 5/(5 - 4) = 5/(5 - 4)

That amounts to:

=> 5/1 = 5/1

As a result, x = 5 is a correct answer to the problem.

In conclusion, only one of the solutions -x = 5 is correct, and the other

-x = 4 is unnecessary because it results in a divide by zero.

To know more about equation visit:

https://brainly.com/question/649785

#SPJ1

Other Questions
Will mark as brainliest!!!What, in your opinion, are the two biggest social problems? Why? What are the social problems you see most often in your community? How do you think social conditions can be improved? What do you think politicians should do? What do you think people should do? Answer these questions in Spanish. trough is 10 ft long and its ends have the shape of isosceles triangles that are 3 ft across at the top and have a height of 1 ft. if the trough is being flled with water at a rate of 12 ft 3 ymin, how fast is the water level rising when the water is 6 inches deep? who or what is the power elite? is it stable? how does an elite view of power pose tension for a democratic society? is conflict between elite groups normal? dillon Please I need help this is For A DRAMA class For your week three discussion question use the lists of Dont and Be Careful listed above and pick three moments/scenes from Some Like it Hot (1959) that would have caused an issue in violation of the Hays Code. Your answers should include and will be graded on:Writing Competency Document should contain 250 words for the complete assignment.A detailed example of each of those 3 moments/scenes listing characters and scene involved.Explain how each particular moment/scene you have chosen would have violated the Hays Code. Be specific and give an example of which Code number would have been involved.Please do not plagiarize. Remember the Academic Dishonesty section discussed in the syllabus! You are conducting an investigation on a suspected compromise. You have noticed several files that you don't recognize. How can you quickly and effectively check if the files have been infected with malware?Submit the files to an open-source intelligence provider like VirusTotalDisassembly the files and conduct static analysis on them using IDA ProRun the Strings tool against each file to identify common malware identifiersScan the files using a local anti-virus/anti-malware engine Please help me on this physics question f(x) = 2x - 1 g(x) = 7x + 8 find (gof) (x) What is the rate of return when 15 shares of Stock A, purchased for $30/share, are sold for $500? The commission on the sale is $6. Rate of Return = [?] % Give your answer as a percent rounded to the nearest tenth. sadie received $54,000 of compensation from her employer and she received $485 of interest from a corporate bond. what is the amount of sadie's gross income from these items? Whats the the sum of x and 2 which of the methods can be used to improve the resolution between two compounds for a liquid separation using a packed chromatography column? which action is nurse's priority when a client with severe chest pain and diaphoresis arrives in the emergency department? In The Masque of the Red Death Poe describes the window panes of the black room as "scarleta deep blood color." What is the effect of this word choice?A. The word choice makes associations with the human body.B. The word choice reminds readers that Prince Prospero is weird.C. The word choice helps create the party like atmosphere.D. The word choice mirrors the deadly plague outside the castle walls. which social media metric is probably the most useful for determining whether a company is increasing its brand awareness on social media? multiple choice total followers on social media platforms total number of likes for social media posts total mentions of the company on social media total sales of products advertised on social media a sandstone bed strikes 140 across a stream. the tresam flows down a narrow gorge with vertical walls. the apparent dip of the bed on the walls of the gorge is 25, 095. what is the true dip of the bed? What is the value of k?127.3 What key words hint that Holmes knows the Counts identity? what effect(s) might a decreased rate of photosynthesis have on cellular respiration in mitochondria? exploraton in math suppose a principal amount of $15,400 is borrowed at a simple interest rate 15% for a period of 13 years. determine the amount of simple interest owed for the use of this loan. round the solution to the nearest cent, if necessary. the amount of simple interest owed is it is very easy for evelyn to find inexpensive inputs for her business. evelyn's supply is therefore likely to be: elastic. perfectly elastic. inelastic. unit-elastic.